LSAT and Law School Admissions Forum

Get expert LSAT preparation and law school admissions advice from PowerScore Test Preparation.

 Administrator
PowerScore Staff
  • PowerScore Staff
  • Posts: 8917
  • Joined: Feb 02, 2011
|
#41658
Complete Question Explanation
(The complete setup for this game can be found here: lsat/viewtopic.php?t=8572)

The correct answer choice is (C)

If G, the hour-long program, is shown third, then we need to refer to Templates 2A and 2B. Due to the high variance in possible solutions inherent to Template 2A, prephrasing the answer to this Could Be True question is virtually impossible. Instead, proceed by the process of elimination: any answer choice that cannot be satisfied in either template will be incorrect.
Answer choice (A): This answer choice is incorrect, because R is not the second program in either template. Either T or S is.

Answer choice (B): This answer choice is incorrect, because R is not the fifth program in either template. In Template 2A, the fifth program must be either S or W. In Template 2B, the fifth program is T.

Answer choice (C): This is the correct answer choice, because S could be the fourth program in Template 2A:
PT65_D11 LG Explanations_game_#4_#21_diagram 1.png
Answer choice (D): This answer choice is incorrect, because T is not the fourth program in either template. T must be either the second or the fifth program.

Answer choice (E): This answer choice is incorrect, because W is not the second program in either template. Either T or S is.
You do not have the required permissions to view the files attached to this post.
 Tw07
  • Posts: 7
  • Joined: Feb 15, 2020
|
#73921
At first I could not figure out why R could not be 5th because it is possible for it to be W-T-G-G-R-S. But I think I missed the fact that the question is considering G as one program, even though it takes two slots. So R being 5th would really be 3:30, not 3. Is that right?
 Adam Tyson
PowerScore Staff
  • PowerScore Staff
  • Posts: 5153
  • Joined: Apr 14, 2011
|
#73948
That's correct, and it is one of the things that makes this game so challenging. Treating that GG block as a single program when counting things like "the third program" and "the second program" and "the fifth program" is crucial! It's also the key to another question in this game, the dreaded Rule Substitution question! Because the GG block has to start at 1:00, or 2:00, or 3:00, it can only be the first, third, or fifth program in the schedule. Check out the answer choices to question 23 and see how that helps!

Get the most out of your LSAT Prep Plus subscription.

Analyze and track your performance with our Testing and Analytics Package.